7
$\begingroup$

I have a ground set $A$ of $n$ elements. I want to create a collection $C$ of $m$ sets of size $k < n$ such that every subset $S\subseteq A$ of size $k$ is covered by at most $t$ sets from $C$ (i.e., every set $S$ is contained in a union of at most $t$ sets from $C$). What is the minimum value of $m$ such that such a collection exists as a function of $t$ (and $n,k$)? For example, for $t=1$ we need all such subsets so $m= {{n} \choose{k}} $, and for $t=n/k$ we can create an arbitrary partition so we need $m=\lceil n/k\rceil$.

In particular, what is $m$ for $t \in \{2,\log(n/k),\sqrt{n/k}\}$?

$\endgroup$
1
  • 2
    $\begingroup$ Have you tried randomly sampling sets for $C$? This feels like it should give a pretty good bound $\endgroup$ Commented Jan 27 at 19:32

1 Answer 1

3
$\begingroup$

For $t=2$ and $1 \le k < n \le 10$, I used integer linear programming to find the minimum values for $m$. The values in red are the only ones that differ from $\lceil n/k \rceil$: \begin{matrix} n \backslash k & 1 & 2 & 3 & 4 & 5 & 6 & 7 & 8 & 9 \\ \hline 2 & 2 \\ 3 & 3 & 2 \\ 4 & 4 & 2 & 2 \\ 5 & 5 & 3 & 2 & 2 \\ 6 & 6 & 3 & 2 & 2 & 2 \\ 7 & 7 & 4 & \color{red}4 & 2 & 2 & 2 \\ 8 & 8 & 4 & \color{red}5 & 2 & 2 & 2 & 2 \\ 9 & 9 & 5 & \color{red}7 & \color{red}5 & 2 & 2 & 2 & 2 \\ 10 & 10 & 5 & \color{red}8 & \color{red}5 & 2 & 2 & 2 & 2 & 2 \\ \end{matrix}

$\endgroup$

You must log in to answer this question.

Start asking to get answers

Find the answer to your question by asking.

Ask question

Explore related questions

See similar questions with these tags.